0
$\begingroup$

Prove that if $f \in L^{1}(A)$ and {${A_{n}}$} is a sequence of measurable subsets of A with $\lim_{n \to \infty}m(A_{n})=0$ then $lim_{n \to \infty} \int_{A_n}f=0$

I know there exists a very similar post for tackling this particular problem but I cannot find the complete proof of it. I would be grateful if someone could help.

$\endgroup$

2 Answers 2

0
$\begingroup$

$\lvert\int f1_{A_n}\,dm\rvert\leq\int\lvert f1_{A_n}\rvert\,dm\leq m(A_n)\int\lvert f\rvert\,dm\to0$ by Holder's inequality.

$\endgroup$
2
  • $\begingroup$ Thanks for the response. Will the same apply if we did not have that f was in $L^1$? For example if we had that f was integrable in $(R,Σ,m)$? $\endgroup$
    – user415301
    Feb 11, 2017 at 19:51
  • $\begingroup$ $f$ being integrable is the same as belonging to $L^1$. $\endgroup$
    – user375366
    Feb 12, 2017 at 0:40
-1
$\begingroup$

It's easy. For $n \in{} \mathbb{N}$, $\int_{A_n} f=\int_{A} f \mathbb{1}_{A_n}$. By Holder's inequallity, we have $\left |{\int_{A} f \mathbb{1}_{A_n}}\right |^2 \leq \int_{A} f^2 \int_{A} \mathbb{1}_{A_n}^2= \int_{A} f^2 \int_{A} \mathbb{1}_{A_n}=m(A_n)\int_{A} f^2 \xrightarrow{ n \to \infty } 0$ since $\lim_{ n \to \infty } m(A_n)=0$ and $f \in L^1(A)$. We conclude $\lim_{ n \to \infty } \int_{A} f \mathbb{1}_{A_n}=0$.

$\endgroup$
2
  • $\begingroup$ If $f \in L^1$, $f$ is not necessarily $L^2$... $\endgroup$
    – GaC
    Feb 11, 2017 at 13:41
  • $\begingroup$ You are right. I ignored that mistake. $\endgroup$
    – asd
    Feb 20, 2017 at 20:56

You must log in to answer this question.